Вы находитесь на странице: 1из 24

OpenStax College Physics Instructor Solutions Manual Chapter 15

CHAPTER 15: THERMODYNAMICS


15.1 THE FIRST LAW OF THERMODYNAMICS

1. What is the change in internal energy of a car if you put 12.0 gal of gasoline into its
tank? The energy content of gasoline is 1.3 108 J/gal . All other factors, such as the
car’s temperature, are constant.

Solution Using the energy content of a gallon of gasoline 1.3 108 J/gal , the energy stored in
12.0 gallons of gasoline is Egas  (1.3 108 J/gal)( 12.0 gal)  1.6 109 J. Therefore, the
internal energy of the car increases by this energy, so that U  1.6 109 J

2. How much heat transfer occurs from a system, if its internal energy decreased by 150
J while it was doing 30.0 J of work?

Solution ΔU  Q  W  Q  ΔU  W  150 J  30.0 J  120 J

Thus, 120 J was lost in the heat.

3. A system does 1.80  108 J of work while 7.50 108 J of heat transfer occurs to the
environment. What is the change in internal energy of the system assuming no other
changes (such as in temperature or by the addition of fuel)?

Solution U  Q  W  7.50 108 J  1.80 108 J   9.30 108 J

4. What is the change in internal energy of a system which does 4.50 10 J of work
5

while 3.00 10 J of heat transfer occurs into the system, and 8.00 10 J of heat
6 6

transfer occurs to the environment?

Solution U  Q  W  (3.00 106 J  8.00 106 J)  4.50 105 J   5.45 106 J

5. Suppose a woman does 500 J of work and 9500 J of heat transfer occurs into the
environment in the process. (a) What is the decrease in her internal energy, assuming
no change in temperature or consumption of food? (That is, there is no other energy
transfer.) (b) What is her efficiency?
OpenStax College Physics Instructor Solutions Manual Chapter 15

Solution (a) ΔU  W  9500 J  500 J  10,000 J   1.0 104 J or  2.39 kcal

Wout W  500 J
(b) Eff     0.0500 , or 5.00%
Ein U  10,000 J

6. (a) How much food energy will a man metabolize in the process of doing 35.0 kJ of
work with an efficiency of 5.00%? (b) How much heat transfer occurs to the
environment to keep his temperature constant? Explicitly show how you follow the
steps found in Problem-Solving Strategies for Thermodynamics.

Solution Wout W
(a) Eff   
Ein U
W (35.0 kJ)(1 kcal/ 4.186 kJ)
U    167.2 kcal  167 kcal
Eff 0.0500

(b) Qwasted  (167.2 kcal)( 0.950)  159 kcal

7. (a) What is the average metabolic rate in watts of a man who metabolizes 10,500 kJ
of food energy in one day? (b) What is the maximum amount of work in joules he can
do without breaking down fat, assuming a maximum efficiency of 20.0%? (c) Compare
his work output with the daily output of a 187-W (0.250-horsepower) motor.

Solution Q 10500 kJ 1000 J 1 day


(a) P      122 W
t 1 day 1 kJ 8.64 10 4 s

W 1000 J
(b) Eff  , so W  Eff  Ein  0.200(10500 kJ)   2.10  10 6 J
Ein 1 kJ

(c) Work done by the motor is


746 W 8.64  10 4 s
W  Pt  (0.250 hp)(1 day)    1.61 10 7 J
1 hp 1 day

Wmotor 1.6110 7 J
Then,   7.67 . Thus the motor produces 7.67 times the work
Wman 2.10 10 6 J
done by the man.
OpenStax College Physics Instructor Solutions Manual Chapter 15

8. (a) How long will the energy in a 1470-kJ (350-kcal) cup of yogurt last in a woman
doing work at the rate of 150 W with an efficiency of 20.0% (such as in leisurely
climbing stairs)? (b) Does the time found in part (a) imply that it is easy to consume
more food energy than you can reasonably expect to work off with exercise?

Solution W Pt
(a) Eff   
U U
Eff  U 0.200(350 kcal)( 4186 J/1 kcal)
t   1.95  10 3 s  32.6 min
P 150 J/s

(b) Since eating an extra 350 kcal cup of yogurt takes a half an hour to burn off, just
one candy bar takes about that same amount of time to work off, so it is
definitely easy to consume more food calories than you can expect to work off
with exercise! That is why it is so important to watch your calorie intake when
you are trying to lose weight.

9. (a) A woman climbing the Washington Monument metabolizes 6.00 10 kJ of food
2

energy. If her efficiency is 18.0%, how much heat transfer occurs to the environment
to keep her temperature constant? (b) Discuss the amount of heat transfer found in
(a). Is it consistent with the fact that you quickly warm up when exercising?

Solution (a) ΔU  0  Q  W or W  Q  Qin  Qout 


W
Qout  Qin  W  Qin  0.180Qin (since Eff   0.180)
Qin
Qout  0.820Qin  0.820(600 kJ)  492 kJ

(b) This amount of heat is consistent with the fact that you warm quickly when
exercising. Since the body is inefficient, the excess heat produced must be
dissipated through sweating, breathing, etc.

15.2 THE FIRST LAW OF THERMODYNAMICS AND SOME SIMPLE


PROCESSES

10. A car tire contains 0.0380 m 3 of air at a pressure of 2.20 105 N/m 2 (about 32 psi).
How much more internal energy does this gas have than the same volume has at zero
gauge pressure (which is equivalent to normal atmospheric pressure)?

Solution Assume V1  V2 , where state 1 is that at 32 psi and state 2 is that at gauge pressure
OpenStax College Physics Instructor Solutions Manual Chapter 15

(1 atm).

P1V PV
P1V  n1 RT  n1  and P2V  n2 RT  n2  2 so that
RT RT
U 1 (3 / 2)n1 RT n P
  1  1
U 2 (3 / 2)n 2 RT n2 P2
3  P2V  3
U2    RT  P2V  1.5(1.013  10 N/m )(0.038 m )  5.77  10 J
5 2 3 3

2  RT  2
P   2.20  10 5 N/m 2 
U 1   1 U 2   (5.77  10 3 J)  1.25  10 4 J
2 
 1.013  10 N/m 
5
 2
P
U 1  U 2  (1.25  10 4 J)  (5.77  10 3 J)  6.77  10 3 J

11. A helium-filled toy balloon has a gauge pressure of 0.200 atm and a volume of 10.0 L.
How much greater is the internal energy of the helium in the balloon than it would be
at zero gauge pressure?

Solution The volume remains constant, so that V1  V2 , and

P1  0.200 atm  Pa  0.200 atm  1.00 atm  1.20 atm , and P2  1.00 atm .

For a monatomic gas:

3 3 PV  3
U2  N 2 kT   2 kT  P2V
2 2  kT  2
3 1.013  10 5 N/m 2  10 3 m 3 
 1 atm  10.0 L    1.52  10 3 J
2 1 atm  1 L 
U 1 3 / 2 N 1 kT N 1 P1V / kT P1
    , so that
U 2 3 / 2 N 2 kT N 2 P2V / kT P2

P   1.20 atm 
U 1   1 U 2    
 1.52  10 J  1.82  10 J
3 3

 2
P  1.00 atm 

and so: U1  U 2  1.82 103 J -1.52 103 J  300 J


OpenStax College Physics Instructor Solutions Manual Chapter 15

12. Steam to drive an old-fashioned steam locomotive is supplied at a constant gauge


pressure of 1.75 10 N/m (about 250 psi) to a piston with a 0.200-m radius. (a) By
6 2

calculating PV , find the work done by the steam when the piston moves 0.800 m.
Note that this is the net work output, since gauge pressure is used. (b) Now find the
amount of work by calculating the force exerted times the distance traveled. Is the

answer the same as in part (a)?

Solution (a) W  PV  PAd  P r 2 d


 (1.75  10 6 N/m 2 )π (0.200 m) 2 (0.800 m)  1.76  10 5 J

(b) W  Fd  PAd  1.76 105 J . Yes, the answer is the same.

13. A hand-driven tire pump has a piston with a 2.50-cm diameter and a maximum stroke
of 30.0 cm. (a) How much work do you do in one stroke if the average gauge pressure
is 2.40 10 N/m (about 35 psi)? (b) What average force do you exert on the piston,
5 2

neglecting friction and gravitational force?

Solution (a) W  PV  P r 2 d  (2.40  105 N/m 2 ) (1.25  10 - 2 m) 2 (0.300 m)  35.3 J

W 35.3 J
(b) W  Fd  F    118 N
d 0.300 m

14. Calculate the net work output of a heat engine following path ABCDA in the figure
below.

Solution 0.5( 0.6  10 6 N/m 2 )  1.0  10 6 N/m 2 


W  area    3

 3.0  10 m  4.5  10 J
3 3

 0.5( 0.4  10 N/m ) 


6 2

15. What is the net work output of a heat engine that follows path ABDA in the figure
above, with a straight line from B to D? Why is the work output less than for path
ABCDA? Explicitly show how you follow the steps in the Problem-Solving Strategies for
Thermodynamics.

Solution   
W  area  0.5 1.6  10 6 N/m 2 3.00  10 3 m 3  2.4  10 3 J

The work done for path ABDA is less than for ABCDA because the environment does
more work on the system in BD than in BCD. We see this diagrammatically because
the area enclosed by ABDA is less than that enclosed by ABCDA.
OpenStax College Physics Instructor Solutions Manual Chapter 15

16. Unreasonable Results What is wrong with the claim that a cyclical heat engine does
4.00 kJ of work on an input of 24.0 kJ of heat transfer while 16.0 kJ of heat transfers to
the environment?

Solution W  4.00 kJ, Qh  24.0 kJ, Qc  16.0 kJ


Qh  Qc  24.0 kJ  16.0 kJ  8.0 kJ  4.0 kJ.
Thus ,W  Qh - Qc

For a cyclical engine, W must equal Qh  Qc . W is not equal to the difference


between the heat input and heat output.

17. (a) A cyclical heat engine, operating between temperatures of 450C and 150C
produces 4.00 MJ of work on a heat transfer of 5.00 MJ into the engine. How much
heat transfer occurs to the environment? (b) What is unreasonable about the engine?
(c) Which premise is unreasonable?  

Solution (a) W  Qh  Qc  Qc  Qh  W  5.00 MJ  4.00 MJ  1.00 MJ

W 4.00 MJ T  Tc 300 K
(b) Eff    0.800, and EffC  h   0.415
Qh 5.00 MJ Th 723 K

It is unreasonable that Eff  EffC .

(c) It is unreasonable that the engine does that much work for the given heat input
for the given hot and cold temperatures.

15.3 INTRODUCTION TO THE SECOND LAW OF THERMODYNAMICS: HEAT


ENGINES AND THEIR EFFICIENCY

20. A certain heat engine does 10.0 kJ of work and 8.50 kJ of heat transfer occurs to the
environment in a cyclical process. (a) What was the heat transfer into this engine? (b)
What was the engine’s efficiency?

Solution (a) U  0  Q  W  W  Q  Qh  Qc
Qh  W  Qc  10.0 kJ  8.5 kJ  18.5 kJ

W 10.0 kJ
(b) Eff    0.541 or 54.1%
Qh 18.5 kJ
OpenStax College Physics Instructor Solutions Manual Chapter 15

21. With 2.56 10 J of heat transfer into this engine, a given cyclical heat engine can do
6

only 1.50  10 J of work. (a) What is the engine’s efficiency? (b) How much heat
5

transfer to the environment takes place?

Solution (a) The efficiency is the work out divided by the heat in:

W 1.50 105 J
Eff    0.0586, or 5.86%
Qh 2.56 10 6 J

(b) The work output is the difference between the heat input and the wasted heat, so
from the first law of thermodynamics:

W  Qh  Qc  Qc  Qh  W  2.56  10 6 J  1.50  10 5 J  2.41 10 6 J

22. (a) What is the work output of a cyclical heat engine having a 22.0% efficiency and
6.00 109 J of heat transfer into the engine? (b) How much heat transfer occurs to the
environment?

Solution W
(a) Eff   W  Eff  Qh  0.220(6.00  10 9 J)  1.32  10 9 J
Qh

(b) W  Qh  Qc  Qc  Qh  W  6.00  10 9 J  1.32  10 9 J  4.68  10 9 J

23. (a) What is the efficiency of a cyclical heat engine in which 75.0 kJ of heat transfer
occurs to the environment for every 95.0 kJ of heat transfer into the engine? (b) How
much work does it produce for 100 kJ of heat transfer into the engine?

Solution W Qh  Qc Q 75.0 kJ
(a) Eff    1 c  1  0.211, or 21.1%
Qh Qh Qh 95.0 kJ

(b) W  Eff  Qh  0.211(100 kJ)  21.1 kJ

24. The engine of a large ship does 2.00 108 J of work with an efficiency of 5.00%. (a)
How much heat transfer occurs to the environment? (b) How many barrels of fuel are
consumed, if each barrel produces 6.00 10 J of heat transfer when burned?
9
OpenStax College Physics Instructor Solutions Manual Chapter 15

Solution (a) W
Qh  Qc  W ; Eff 
Qh
W
 Qc  W 
Eff
 1   1 
Qc    1W    1(2.00  10 8 J)  3.80  10 9 J
 Eff   0.0500 

(b) Let N equal the number of barrels of fuel consumed.

Qh Qc  W 3.80 109 J  2.00 108 J


N    0.667 barrels
6.00 109 J 6.00 109 J 6.00 109 J

25. (a) How much heat transfer occurs to the environment by an electrical power station
that uses 1.25  10 J of heat transfer into the engine with an efficiency of 42.0%? (b)
14

What is the ratio of heat transfer to the environment to work output? (c) How much
work is done?

Solution Qc
(a) Eff  1   Qc  ( 1  Eff )Qh  (1  0.420)(1.25  1014 J)  7.25  1013 J
Qh

Qc Qc 7.25  1013 J
(b)    1.38
W Qh  Qc 1.25  1014 J  7.25  1013 J

(c) W  Qh  Qc  1.25  1014 J  0.725  1014 J  5.25  1013 J

26. Assume that the turbines at a coal-powered power plant were upgraded, resulting in
an improvement in efficiency of 3.32%. Assume that prior to the upgrade the power
station had an efficiency of 36% and that the heat transfer into the engine in one day
is still the same at 2.50 10 J . (a) How much more electrical energy is produced due
14

to the upgrade? (b) How much less heat transfer occurs to the environment due to the
upgrade?

Solution (a) Efficiency increased to 36%  3.32%  39.32% . Additional output will be
0.0332Qin  = 0.0332(2.50  1014 J/d) = 8.3  1012 J/d .
(b) Heat transfer to the environment will be  8.3 1012 J, where the negative sign
indicates a reduction in heat transfer to the environment.
OpenStax College Physics Instructor Solutions Manual Chapter 15

27. This problem compares the energy output and heat transfer to the environment by
two different types of nuclear power stations—one with the normal efficiency of
34.0%, and another with an improved efficiency of 40.0%. Suppose both have the
same heat transfer into the engine in one day, 2.50 1014 J . (a) How much more
electrical energy is produced by the more efficient power station? (b) How much less
heat transfer occurs to the environment by the more efficient power station? (One
type of more efficient nuclear power station, the gas-cooled reactor, has not been
reliable enough to be economically feasible in spite of its greater efficiency.)

Solution W1 W
(a) Eff1  ; Eff 2  2
Qh Qh
W2  W1  Qh ( Eff 2  Eff1 )  (2.50  1014 J)( 0.400  0.340)  1.50  1013 J

(b) Qh  W1  Qc1  W2  Qc2  Qc2  Qc1  (W2  W1 )   1.50  1013 J

15.4 CARNOT’S PERFECT HEAT ENGINE: THE SECOND LAW OF


THERMODYNAMICS RESTATED

28. A certain gasoline engine has an efficiency of 30.0%. What would the hot reservoir
temperature be for a Carnot engine having that efficiency, if it operates with a cold
reservoir temperature of 200C?

Solution Tc Tc 473 K
EffC  1   Th    676 K  403C
Th  1  EffC 1  0.300

29. A gas-cooled nuclear reactor operates between hot and cold reservoir temperatures of
700C and 27.0C. (a) What is the maximum efficiency of a heat engine operating
between these temperatures? (b) Find the ratio of this efficiency to the Carnot
efficiency of a standard nuclear reactor (found in Example 15.4.)
 
Solution Th  Tc 973 K  300 K
(a) EffC    0.692 , or 69.2%
Th 973 K

0.692
(b)  1.45
0.476
OpenStax College Physics Instructor Solutions Manual Chapter 15

30. (a) What is the hot reservoir temperature of a Carnot engine that has an efficiency of
42.0% and a cold reservoir temperature of 27.0C ? (b) What must the hot reservoir
temperature be for a real heat engine that achieves 0.700 of the maximum efficiency,
but still has an efficiency of 42.0% (and a cold reservoir at 27.0C )? (c) Does your
answer imply practical limits to the
 efficiency of car gasoline engines?

Solution Tc Tc 300.0 K 
(a) EffC  1   Th    517.2 K  244C
Th 1  EffC 1  0.420

0.420
(b) 0.700 Eff C  0.420  Eff C   0.600,
0.700
Tc 300 K
so that Th    750 K  477C
1  Eff C 1  0.600

(c) Yes, since automobiles engines cannot get too hot without overheating, their
efficiency is limited.

31. Steam locomotives have an efficiency of 17.0% and operate with a hot steam
temperature of 425C. (a) What would the cold reservoir temperature be if this were
a Carnot engine? (b) What would the maximum efficiency of this steam engine be if its
cold reservoir temperature were 150C ?

Solution Tc
(a) EffC  1   Tc  ( 1  EffC )Th  (1  0.170)(698 K)  579 K  306C
Th 

Tc 423K
(b) EffC  1   1  0.394 , or 39.4%
Th 698K

32. Practical steam engines utilize 450C steam, which is later exhausted at 270C. (a)
What is the maximum efficiency that such a heat engine can have? (b) Since 270C
steam is still quite hot, a second steam engine is sometimes operated using the
exhaust of the first. What
 is the maximum efficiency of the second engine if its
exhaust has a temperature of 150C? (c) What is the overall efficiency
 of the two
engines? (d) Show that this is the same efficiency as a single Carnot engine operating
between 450C and 150C. Explicitly show how you follow the steps in the Problem-
Solving Strategies forThermodynamics.

Solution   T 543 K
(a) EffC1  1  c  1   0.249 or 24.9%
Th 723 K
OpenStax College Physics Instructor Solutions Manual Chapter 15

423 K
(b) Eff C2  1   0.221 or 22.1%
543 K

(c) Eff C1  W1  Qh1  Qc1 , Qc1  Qh1 (1  Eff )and


C1
Qh1 Qh1
Qc2  Qh2 (1  Eff C2 )  Qc1 (1  Eff C2 )  Qh1 (1  Eff C1 )(1  Eff C2 )
W Q  Qc2 Qh1 1  (1  Eff C1 )(1  Eff C2 )
Eff tot   h1   1  (1  Eff C1 )(1  Eff C2 )
Qh1 Qh1 Qh1
Eff tot  1  (1  0.249)(1  0.221)  0.415, or 41.5%

Tc 423 K
(d) EffC  1   1  0.415 or 41.5%
Th 723 K

33. A coal-fired electrical power station has an efficiency of 38%. The temperature of the
steam leaving the boiler is 550C . What percentage of the maximum efficiency does
this station obtain? (Assume the temperature of the environment is 20C .)

Solution Tc 293
Carnot efficiency EffC  1   1  0.644
Th 823 

38.0
So the power plant achieves  0.59  59% of the maximum efficiency.
64.4

34. Would you be willing to financially back an inventor who is marketing a device that
she claims has 25 kJ of heat transfer at 600 K, has heat transfer to the environment at
300 K, and does 12 kJ of work? Explain your answer.

Solution The heat transfer to the cold reservoir is Qc  Qh  W  25 kJ  12 kJ  13 kJ so the


Q 13 kJ
efficiency is Eff  1  c  1   0.48 .
Qh 25 kJ

Tc 300 K
The Carnot efficiency is EffC  1   1  0.50 .
Th 600 K

The actual efficiency is 96% of the Carnot efficiency, which is much higher than the
best-ever achieved of about 70%, so her scheme is likely to be fraudulent.
OpenStax College Physics Instructor Solutions Manual Chapter 15

35. Unreasonable Results (a) Suppose you want to design a steam engine that has heat
transfer to the environment at 270C and has a Carnot efficiency of 0.800. What
temperature of hot steam must you use? (b) What is unreasonable about the
temperature? (c) Which premise is unreasonable?

Solution Th  Tc Tc 543.15 K
(a) EffC   Th    2716 K  2.72  10 3 K
Th 1  EffC 1  0.800

(b) The temperature is too high. (Iron melts at 1809 K.)

(c) The exhaust temperature Tc is too high, as is the assumed efficiency.

36. Unreasonable Results Calculate the cold reservoir temperature of a steam engine that
uses hot steam at 450C and has a Carnot efficiency of 0.700. (b) What is
unreasonable about the temperature? (c) Which premise is unreasonable?

Solution T  Tc
(a) Eff C  h  Tc  Th (1  Eff C )
Th
 723 K(1  0.700)  216.9 K  56.25C   56.3C

(b) The temperature is too cold for the output of a steam engine (the local
environment). It is below the freezing point of water.

(c) The assumed efficiency is too high.

15.5 APPLICATIONS OF THERMODYNAMICS: HEAT PUMPS AND


REFRIGERATORS

37. What is the coefficient of performance of an ideal heat pump that has heat transfer
from a cold temperature of 25.0C to a hot temperature of 40.0C?

Solution 1 Th 313 K
COPhp     4.82

EffC Th  Tc 313 K  248 K 

38. Suppose you have an ideal refrigerator that cools an environment at 20.0C and has
heat transfer to another environment at 50.0C . What is its coefficient of
performance?


OpenStax College Physics Instructor Solutions Manual Chapter 15

Solution 1 Th T  Tc Tc 253 K
COPref  COPhp  1  1   h    3.61
EffC Th  Tc Th  Tc Th  Tc 70.0 K

39. What is the best coefficient of performance possible for a hypothetical refrigerator
that could make liquid nitrogen at 200C and has heat transfer to the environment
at 35.0C?

Solution Tc .0 K
73
COPref    0.311
 Th  Tc 308 K - 73.0 K

40. In a very mild winter climate, a heat pump has heat transfer from an environment at
5.00C to one at 35.0C. What is the best possible coefficient of performance for
these temperatures? Explicitly show how you follow the steps in Problem-Solving
Strategies for Thermodynamics.
 
Solution Th 308 K
COPhp    10.3
Th  Tc 308 K - 278 K

41. (a) What is the best coefficient of performance for a heat pump that has a hot
reservoir temperature of 50.0C and a cold reservoir temperature of 20.0C ? (b)
How much heat transfer occurs into the warm environment if 3.60 10 J of work (
7

10.0 kW h) is put into it? (c) If the cost of this work input is 10.0 cents/kW h , how

does its cost compare with the direct heat transfer achievedby burning natural gas at
a cost of 85.0 cents per therm. (A therm is a common unit of energy for natural gas
and equals 1.055 10 J .)
8
 

Solution Th 323 K
(a) COPhp    4.614  4.61
Th  Tc 70.0 K

W W
(b) Eff   Qh   COPhpW
Qh Eff

 
 4.614 3.60  10 7 J  1.661  10 8 J 
1 kcal
4186 J
 3.97  10 4 kcal

 1 kW  h 
(c) For the heat pump: (3.60  10 7 J) 10.0 ¢/kW  h   $1.00
 3.60  10 J
6

To transfer the same amount of heat using natural gas:


OpenStax College Physics Instructor Solutions Manual Chapter 15

1.66110 J  1.055


8 1 therm 
 10 J
85.0 ¢/therm   $1.34
 
8

To transfer 1.66  10 J , the heat pump costs $1.00, natural gas costs $1.34.
8

42. (a) What is the best coefficient of performance for a refrigerator that cools an
environment at 30.0C and has heat transfer to another environment at 45.0C?
(b) How much work in joules must be done for a heat transfer of 4186 kJ from the cold
environment? (c) What is the cost of doing this if the work costs 10.0 cents per
3.60 
106 J (a kilowatt-hour)? (d) How many kJ of heat transfer occurs
 into the warm
environment? (e) Discuss what type of refrigerator might operate between these
temperatures.

Solution Qc 1 Th
(a) COPref   COPhp  1  1  1
W EffC Th  Tc
Th  (Th  Tc ) Tc 243 K
    3.24
Th  Tc Th  Tc 318 K  243 K

QC QC 1000 kcal
(b) COPref  W    308.6 kcal  309 kcal
W COPref 3.24

 4186 J  10.0¢ 
(c) cost  308.6 kcal    3.59¢
 1 kcal  3.60  10 J 
6

(d) W  Qh  Qc  Qh  W  Qc  309 kcal  1000 kcal  1309 kcal


 4.186 kJ 
 1309 kcal     5479 kJ
 1 kcal 

(e) The inside of this refrigerator (actually freezer) is at  22F (30.0C) , so this
probably is a commercial meat packing freezer. The exhaust is generally vented to
the outside, so as to not heat the building too much.

43. Suppose you want to operate an ideal refrigerator with a cold temperature of
10.0C, and you would like it to have a coefficient of performance of 7.00. What is
the hot reservoir temperature for such a refrigerator?


Solution Tc Tc 263 K
COPref   Th   Tc   263 K  300.57 K  27.6C
Th  Tc COPref 7.00
OpenStax College Physics Instructor Solutions Manual Chapter 15

44. An ideal heat pump is being considered for use in heating an environment with a
temperature of 22.0C . What is the cold reservoir temperature if the pump is to have
a coefficient of performance of 12.0?

Solution  Th
COPhp  
Th  Tc
Th  1   1 
Tc  Th   Th 1   295.15 K1    270.55 K   2.60C
COPhp    12.0 
 COPhp 

45. A 4-ton air conditioner removes 5.06 107 J (48,000 British thermal units) from a cold
environment in 1.00 h. (a) What energy input in joules is necessary to do this if the air
conditioner has an energy efficiency rating ( EER ) of 12.0? (b) What is the cost of
doing this if the work costs 10.0 cents per 3.60 10 J (one kilowatt-hour)? (c) Discuss
6

whether this cost seems realistic. Note that the energy efficiency rating ( EER ) of an

air conditioner or refrigerator is defined to be the number of British thermal units of
heat transfer from a cold environment per hour divided by the watts of power input.

Solution Qc
(a) EER  
W
Q 5.06  10 7 J
W c   1.44  10 7 J
EER (12.0 Btu/h  W)(1.055  10 3 J/Btu)( 1 h/ 3600 s)

10.0¢
(b) cost  1.439  10 7 J   40.0¢
3.60  10 6 J

(c) This cost seems quite realistic, since it says that running an air conditioner all day
would cost $9.59 (if it ran continuously).

46. Show that the coefficients of performance of refrigerators and heat pumps are related
by COPref  COPhp  1 . Start with the definitions of the COPs and the conservation of
energy relationship between Qh , Qc , and W .

Solution Qh Qc
Qh  Qc  W    1,
W W
Q Q
since COPhp  h and COPref  c , we have
W W
COPhp  COPref  1  COPref  COPhp  1
OpenStax College Physics Instructor Solutions Manual Chapter 15

15.6 ENTROPY AND THE SECOND LAW OF THERMODYNAMICS:


DISORDER AND THE UNAVAILABILITY OF ENERGY

47. (a) On a winter day, a certain house loses 5.00 108 J of heat to the outside (about
500,000 Btu). What is the total change in entropy due to this heat transfer alone,
assuming an average indoor temperature of 21.0C and an average outdoor
temperature of 5.00C ? (b) This large change in entropy implies a large amount of
energy has become unavailable to do work. Where do we find more energy when such
energy is lost to us? 

Solution Q
(a) Use ΔS  to calculate the change in entropy, remembering to use
T
temperatures in Kelvin:

 Qh Qc 1 1 
ΔS    Q   
  5.00  108 J 
1

1 
  9.78  10 J/K
4

Th Tc  Tc Th   278 K 294 K 

(b) In order to gain more energy, we must generate it from things within the house,
like a heat pump, human bodies, and other appliances. As you know, we use a lot
of energy to keep our houses warm in the winter, because of the loss of heat to
the outside.

48. On a hot summer day, 4.00 10 J of heat transfer into a parked car takes place,
6

increasing its temperature from 35.0C to 45.0C. What is the increase in entropy of
the car due to this heat transfer alone?

Solution Q 4.00  10 6 J 


ΔS    1.28  10 4 J/K
T 313 K

49. A hot rock ejected from a volcano’s lava fountain cools from 1100C to 40.0C, and
its entropy decreases by 950 J/K. How much heat transfer occurs from the rock?

Solution Q 1373  313  


ΔS   Q  TΔS , since T   843 K,
T 2
Q  (843 K)(950 J/K)  8.01  10 J
5

50. When 1.60  10 J of heat transfer occurs into a meat pie initially at 20.0C, its
5

entropy increases by 480 J/K. What is its final temperature?


OpenStax College Physics Instructor Solutions Manual Chapter 15

Solution Q Q 2Q
ΔS   
T (Ti  Tf ) / 2 Ti  Tf
2Q 2(1.60 10 5 J)
Tf   Ti   293 K  374 K  101C
S 480 J/K

51. The Sun radiates energy at the rate of 3.80 10 W from its 5500C surface into
26

dark empty space (a negligible fraction radiates onto Earth and the other planets).
The effective temperature of deep space is 270C . (a) What is the increase in
entropy in one day due to this heat transfer? (b) How
 much work is made unavailable?

Solution  1 1  Q  1  1
(a) ΔS  Q    t   
 Tc Th  t  Tc Th 

  
 1
ΔS  3.80  10 26 J/s 8.64  10 4 s   
1 
  1.042  10 J/K
31

 3.15 K 5773 K 
 1.04  10 J/K
31

(b) Wunavail  ΔS  T0  (1.042  10 31 J/K)(3.15 K)  3.28  10 31 J

52. (a) In reaching equilibrium, how much heat transfer occurs from 1.00 kg of water at
40.0C when it is placed in contact with 1.00 kg of 20.0C water? (b) What is the
change in entropy due to this heat transfer? (c) How much work is made unavailable,
taking the lowest temperature to be 20.0C? Explicitly show how you follow the steps
 in the Problem-Solving Strategies for Entropy.


Solution (a) Tf  30C, since there 


are equal masses of water in the 20C and 40C
reservoirs.

Q  mcT  (1.00 kg)(4186 J/kg  C)(10.0C)  4.186  10 4 J  4.19  10 4 J

Q 4.186 10 4 J
(b) For the hot water: ΔS h    135.91 J/K .
T 308 K

Q 4.186 10 4 J
For the cold water: ΔS c    140.47 J/K , so the total change in
T 298 K
entropy is ΔS tot  140.47 J/K  (135.91 J/K)  4.56 J/K

(c) Wunavail  ΔS  T0  (4.56 J/K)(293 K)  1.34  10 3 J


OpenStax College Physics Instructor Solutions Manual Chapter 15

53. What is the decrease in entropy of 25.0 g of water that condenses on a bathroom
mirror at a temperature of 35.0C, assuming no change in temperature and given the
latent heat of vaporization to be 2450 kJ/kg?

Solution Q  mLv   (25.0  10 3 kg)( 2450  10 3 J/kg)


S      199 J/K
T T 308 K

The entropy of the water decreases by 199 J/K when it condenses.

54. Find the increase in entropy of 1.00 kg of liquid nitrogen that starts at its boiling
temperature, boils, and warms to 20.0C at constant pressure.

Solution Qb mLv (1.00 kg)( 201  10 3 J/kg)


S boil      2.599  10 3 J/K
T T 77.35 K
Q mc p T 77.35  293.15
S warm  w  and since T   185.25 K, we see that
T T 2
(1.00 kg)(1040 J/kg  C)(215.8C)
S warm   1.212  10 3 J/K
185.25 K
S tot  S boil  S warm  3.81  10 3 J/K

55. A large electrical power station generates 1000 MW of electricity with an efficiency of
35.0%. (a) Calculate the heat transfer to the power station, Qh , in one day. (b) How
much heat transfer Qc occurs to the environment in one day? (c) If the heat transfer
in the cooling towers is from 35.0C water into the local air mass, which increases in
temperature from 18.0C to 20.0C , what is the total increase in entropy due to this
heat transfer? (d) How much energy becomes unavailable to do work because of this
increase in entropy,assuming an 18.0C lowest temperature? (Part of Qc could be
utilized tooperate heat
 engines or for simply heating the surroundings, but it rarely
is.)

Solution (a) W
Eff  
Qh
W Pt (1000  10 6 W)(8.64  10 4 s)
Qh     2.469  1014 J  2.47  1014 J
Eff Eff 0.350

(b) Qh  Qc  W  Qc  Qh  W
Qc  2.469  1014 J  (1000  10 6 W)(8.64  10 4 s)  1.605  1014 J  1.60  1014 J
OpenStax College Physics Instructor Solutions Manual Chapter 15

1 1 18C  20C
(c) S  Qc    and since Tc   19C  292 K,
 Tc Th  2
 1 1 
S  (1.605  1014 J)    2.85  10 J/K
10

 292 K 308 K 

(d) Wunavail  S  T0  (2.85  1010 J/K)( 291 K)  8.29  1012 J

56. (a) How much heat transfer occurs from 20.0 kg of 90.0C water placed in contact
with 20.0 kg of 10.0C water, producing a final temperature of 50.0C ? (b) How
much work could a Carnot engine do with this heat transfer, assuming it operates
between two reservoirs at constant temperatures of 90.0C and 10.0C ? (c) What
increase in entropy is produced by mixing 20.0 kg of 90.0C water with 20.0 kg of
10.0C water? (d) Calculate the amount of work made unavailable by this mixing
using a low temperature of 10.0C , and compare
 it withthe work done by the Carnot
engine. Explicitly show how you follow the steps in the Problem-Solving Strategies for
Entropy. (e) Discuss how everyday processes make increasingly more energy
unavailable to do work, as implied by this problem.

Solution (a) Qh  mcT  (20.0 kg)( 4186 J/kg  C)(90.0C  50C)


 3.349  10 6 J  3.35  10 6 J

Th  Tc 363 K  283 K
(b) Eff C    0.2204, so that
Th 263 K
W  Qh  Eff C  (3.349  10 6 J)( 0.2204)  7.38  10 5 J

 Qh  3.349  10 6 J
(c) S h    9.760  10 3 J/K ,
Th 343 K
Qc  3.349  10 6 J
S c    1.183  10 4 J/K, so that
Tc 303 K
S tot  S h  S c  (9.760  10 3 J/K)  (1.183  10 4 J/K)  2.07  10 3 J/K

(d) Wunavail  S  T0  (2.07  103 J/K)( 283 K)


 5.85  105 J  0.793 of work done by engine (7.38  105 J)

(e) This problem says that 79.3% of the work done by the engine is unavailable, so in
everyday processes, even more energy would be unavailable to do work, since
everyday processes are not as efficient as Carnot systems.
OpenStax College Physics Instructor Solutions Manual Chapter 15

15.7 STATISTICAL INTERPRETATION OF ENTROPY AND THE SECOND LAW


OF THERMODYNAMICS: THE UNDERLYING EXPLANATION

57. Using Table 15.4, verify the contention that if you toss 100 coins each second, you can
22
expect to get 100 heads or 100 tails once in 210 years; calculate the time to two-
digit accuracy.

Solution It should happen twice in every 1.27 1030 s, or once in every 6.35  10 29 s.

 1 h  1 d  1 y 
(6.35  10 29 s)     2.0  10 y
22

 3600 s  24 h  365.25 d 

58. What percent of the time will you get something in the range from 60 heads and 40
tails through 40 heads and 60 tails when tossing 100 coins? The total number of
microstates in that range is 1.22 10 . (Consult Table 15.4.)
30

Solution 1.22  10 30
 0.961  96.1%
1.27  10 30

59. (a) If tossing 100 coins, how many ways (microstates) are there to get the three most
likely macrostates of 49 heads and 51 tails, 50 heads and 50 tails, and 51 heads and
49 tails? (b) What percent of the total possibilities is this? (Consult Table 15.4.)

Solution (a)
H T No. of ways

49 51 9.9  10 28

50 50 1.0  10 29

51 49 9.9  1028

Total = 2.98 1029  3.0 1029

total # of ways to get 3 macrostate s 3.0  10 29


(b) %    0.236  24%
total # of ways 1.27  10 30
OpenStax College Physics Instructor Solutions Manual Chapter 15

60. (a) What is the change in entropy if you start with 100 coins in the 45 heads and 55
tails macrostate, toss them, and get 51 heads and 49 tails? (b) What if you get 75
heads and 25 tails? (c) How much more likely is 51 heads and 49 tails than 75 heads
and 25 tails? (d) Does either outcome violate the second law of thermodynamics?

Solution W 
(a) S  k ln Wf  k ln Wi  k ln  f 
 Wi 
 9.9  10 28 
 1.38  10  23 J/K ln    6.7  10  24 J/K
28 
 6.1  10 

 2.4 10 23 

(b) S  1.38 10 23

J/K ln  28 
   1.7 10 22 J/K
 6.110 

9.9  10 28
(c)  4.1  10 5 times more likely
2.4  10 23

(d) No, entropy decreases are possible, but unlikely. Also, in the second case, S is a
small negative number, indicating that a spontaneous fluctuation in the system
that decreases the entropy is not highly unlikely.

61. (a) What is the change in entropy if you start with 10 coins in the 5 heads and 5 tails
macrostate, toss them, and get 2 heads and 8 tails? (b) How much more likely is 5
heads and 5 tails than 2 heads and 8 tails? (Take the ratio of the number of
microstates to find out.) (c) If you were betting on 2 heads and 8 tails would you
accept odds of 252 to 45? Explain why or why not.

Solution W 
(a) S  k ln Wf  k ln Wi  k ln  f    45 
  1.38  10 23 J/K ln   23
   2.38  10 J/K
 Wi   252 

252
(b)  5.6 times more likely
45

(c) If you were betting on 2 heads and 8 tails, the odds of breaking even are 252 to 45,
so on average you would break even. Therefore, no, you would not bet on odds of
252 to 45.
OpenStax College Physics Instructor Solutions Manual Chapter 15

62. (a) If you toss 10 coins, what percent of the time will you get the three most likely
macrostates (6 heads and 4 tails, 5 heads and 5 tails, 4 heads and 6 tails)? (b) You can
realistically toss 10 coins and count the number of heads and tails about twice a
minute. At that rate, how long will it take on average to get either 10 heads and 0
tails or 0 heads and 10 tails?

Solution  210  252  210 


(a) %6H, 4T or 5H, 5T, or 4H 6T   100     65.6%
 1024 

2 1
(b) The probability of 10H, 0T or 0H, 10T is  . Thus, on average it will take
1024 512
512 trials or, at a rate of 2 trials/min, 256 min. In hours, 256 min = 4.27 h

63. (a) Construct a table showing the macrostates and all of the individual microstates for
tossing 6 coins. (Use Table 15.5 as a guide.) (b) How many macrostates are there? (c)
What is the total number of microstates? (d) What percent chance is there of tossing
5 heads and 1 tail? (e) How much more likely are you to toss 3 heads and 3 tails than
5 heads and 1 tail? (Take the ratio of the number of microstates to find out.)
OpenStax College Physics Instructor Solutions Manual Chapter 15

Solution (a) 6 coin toss

Macrostate Individual Microstates Number of


Microstates

H T

6 0 HHHHHH 1

5 1 HHHHHT HHHHTH HHHTHH HHTHHH HTHHHH 6


THHHHH

4 2 TTHHHH THTHHH THHTHH THHHTH THHHHT 15


HTTHHH HTHTHH HTHHTH HTHHHT HHTTHH
HHTHTH HHTHHT HHHTTH HHHTHT HHHHTT

3 3 TTTHHH TTHTHH TTHHTH TTHHHT THTTHH 20

THTHTH THTHHT THHTTH THHTHT THHHTT

HTTTHH HTTHTH HTTHHT HTHTTH HTHTHT

HTHHTT HHTTTH HHTTHT HHTHTT HHHTTT

2 4 Entries opposite of 4-2 15

1 5 TTTTTH TTTTHT TTTHTT TTHTTT THTTTT HTTTTT 6

0 6 TTTTTT 1

64

(b) 7

(c) 64
OpenStax College Physics Instructor Solutions Manual Chapter 15

6
(d)  0.0938  9.38%
64

20 1
(e)  3 times more likely
6 3

64. In an air conditioner, 12.65 MJ of heat transfer occurs from a cold environment in 1.00
h. (a) What mass of ice melting would involve the same heat transfer? (b) How many
hours of operation would be equivalent to melting 900 kg of ice? (c) If ice costs 20
cents per kg, do you think the air conditioner could be operated more cheaply than by
simply using ice? Describe in detail how you evaluate the relative costs.

Solution Q 12.65  106 J


(a) Q  mLf  m    37.87 kg  37.9 kg
Lf 334  103 J/kg

 4.448 N  1 h 
(b) t  2000 lb 
1
 
2 
  23.97 h  24.0 h
 1 lb 9.80 m/s  37.87 kg 

2000 lb 7¢
(c) For ice: cost/day =   14000 ¢/day  $140/day.
day 1 lb

From Problem 15.45b, we can calculate the cost of running air conditioner:
12.65  106 J 10.0¢ 24 h
   843 ¢/day  $8.43/day
1h 3.6  10 J 1 day
6

Clearly, the air conditioner could be operated more cheaply.

This file is copyright 2016, Rice University. All Rights Reserved.

Вам также может понравиться